Difference between revisions of "2012 AMC 12B Problems/Problem 21"
Line 8: | Line 8: | ||
\qquad\textbf{(E)}\ 21\sqrt{6}</math> | \qquad\textbf{(E)}\ 21\sqrt{6}</math> | ||
− | ==Solution== | + | ==Solution (Long)== |
− | Extend <math>AF</math> and <math>YE</math> so that they meet at <math>G</math>. Then <math>\angle FEG=\angle GFE=60^{\circ}</math>, so <math>\angle FGE=60^{\circ}</math> and therefore <math>AB</math> is parallel to <math>YE</math>. Also, since <math>AX</math> is parallel and equal to <math>YZ</math>, we get <math>\angle BAX = \angle ZYE</math>, hence <math>\triangle ABX</math> is congruent to <math>\triangle </ | + | Extend <math>AF</math> and <math>YE</math> so that they meet at <math>G</math>. Then <math>\angle FEG=\angle GFE=60^{\circ}</math>, so <math>\angle FGE=60^{\circ}</math> and therefore <math>AB</math> is parallel to <math>YE</math>. Also, since <math>AX</math> is parallel and equal to <math>YZ</math>, we get <math>\angle BAX = \angle ZYE</math>, hence <math>\triangle ABX</math> is congruent to <math>\triangle YEZ</math>. We now get <math>YE=AB=40</math>. |
− | Let < | + | Let <math>a_1=EY=40</math>, <math>a_2=AF</math>, and <math>a_3=EF</math>. |
− | Drop a perpendicular line from < | + | Drop a perpendicular line from <math>A</math> to the line of <math>EF</math> that meets line <math>EF</math> at <math>K</math>, and a perpendicular line from <math>Y</math> to the line of <math>EF</math> that meets <math>EF</math> at <math>L</math>, then <math>\triangle AKZ</math> is congruent to <math>\triangle ZLY</math> since <math>\angle YLZ</math> is complementary to <math>\angle KZA</math>. Then we have the following equations: |
<cmath>\frac{\sqrt{3}}{2}a_2 = AK=ZL = ZE+\frac{1}{2} a_1</cmath> | <cmath>\frac{\sqrt{3}}{2}a_2 = AK=ZL = ZE+\frac{1}{2} a_1</cmath> | ||
Line 26: | Line 26: | ||
<cmath>a_2=82-40=42.</cmath> | <cmath>a_2=82-40=42.</cmath> | ||
− | So, we can now use the law of cosines in < | + | So, we can now use the law of cosines in <math>\triangle AGY</math>: |
− | <cmath> 2AZ^2 = AY^2 = AG^2 + YG^2 - 2AG\cdot YG \cdot \cos 60^{\circ} = (a_2+a_3)^2 + (a_1+a_3)^2 - (a_2+a_3)(a_1+a_3)</cmath> | + | <cmath> 2AZ^2 = AY^2 = AG^2 + YG^2 - 2AG\cdot YG \cdot \cos 60^{\circ}</cmath> |
− | <cmath> = (41\sqrt{3}+1)^2 + (41\sqrt{3}-1)^2 - (41\sqrt{3}+1)(41\sqrt{3}-1) = 6 \cdot 41^ | + | <cmath> = (a_2+a_3)^2 + (a_1+a_3)^2 - (a_2+a_3)(a_1+a_3)</cmath> |
+ | <cmath> = (41\sqrt{3}+1)^2 + (41\sqrt{3}-1)^2 - (41\sqrt{3}+1)(41\sqrt{3}-1)</cmath> | ||
+ | <cmath> = 6 \cdot 41^2 + 2 - 3 \cdot 41^2 + 1 = 3 (41^2 + 1) = 3\cdot 1682</cmath> | ||
<cmath> AZ^2 = 3 \cdot 841 = 3 \cdot 29^2</cmath> | <cmath> AZ^2 = 3 \cdot 841 = 3 \cdot 29^2</cmath> | ||
− | Therefore < | + | Therefore <math>AZ = 29\sqrt{3} ... \framebox{A}</math> |
Revision as of 22:17, 4 December 2012
Problem
Square is inscribed in equiangular hexagon with on , on , and on . Suppose that , and . What is the side-length of the square?
Solution (Long)
Extend and so that they meet at . Then , so and therefore is parallel to . Also, since is parallel and equal to , we get , hence is congruent to . We now get .
Let , , and .
Drop a perpendicular line from to the line of that meets line at , and a perpendicular line from to the line of that meets at , then is congruent to since is complementary to . Then we have the following equations:
The sum of these two yields that
So, we can now use the law of cosines in :
Therefore